Change of varibles in integrals (More than 1 question)

  • Thread starter Thread starter athrun200
  • Start date Start date
  • Tags Tags
    Change Integrals
Click For Summary
The discussion centers on the challenges of integrating variables in spherical coordinates, specifically integrating θ from 0 to 2π and φ from 0 to π to generate a sphere. Participants highlight that the limits for θ integration are incorrect, as sin θ is positive for 0 < θ < π and negative for π < θ < 2π. Additionally, there are questions about integrating functions that involve both r and θ, with suggestions to use the substitution r = 2a cos(θ) to facilitate the integration. The density of the circular lamina is also discussed, defined as ρ = Mπa², where M represents mass per unit area. Clarifications on the limits of integration for θ are necessary for accurate results.
athrun200
Messages
275
Reaction score
0

Homework Statement


attachment.php?attachmentid=37253&stc=1&d=1310913793.jpg



Homework Equations





The Attempt at a Solution


Why I can't integrate\theta from 0 to 2\pi? Then integrate \varphi from 0 to \pi. It seems it can also generate a sphere.

attachment.php?attachmentid=37254&stc=1&d=1310913793.jpg
 

Attachments

  • 3.jpg
    3.jpg
    15.3 KB · Views: 452
  • 1.jpg
    1.jpg
    34.4 KB · Views: 614
Physics news on Phys.org

Homework Statement


I have questions on d and e

attachment.php?attachmentid=37255&stc=1&d=1310914074.jpg



Homework Equations





The Attempt at a Solution


I don't know how to integrate these functions

attachment.php?attachmentid=37256&stc=1&d=1310914074.jpg


attachment.php?attachmentid=37257&stc=1&d=1310914074.jpg
 

Attachments

  • 6.jpg
    6.jpg
    15.8 KB · Views: 455
  • 5.jpg
    5.jpg
    17.7 KB · Views: 438
  • 4.jpg
    4.jpg
    28.3 KB · Views: 447
athrun200 said:
...
Why I can't integrate\theta from 0 to 2\pi? Then integrate \varphi from 0 to \pi. It seems it can also generate a sphere.

For 0 < θ < π sin θ is positive, for π < θ < 2π sin θ is negative.
 
athrun200 said:

Homework Statement


I have questions on d and e

attachment.php?attachmentid=37255&stc=1&d=1310914074.jpg


I don't know how to integrate these functions
For (d) : You have the wrong limits for the θ integration.

Also, the density is given by ρ = M π a2, where M is the mass of the circular lamina and assumes that ρ is the mass per unit area.

For (e): Your integral has both r & θ in it.

I suggest using r = 2a cos (θ) to find dr/dθ . What are the limits of θ for this integral ?
 
Last edited:
Question: A clock's minute hand has length 4 and its hour hand has length 3. What is the distance between the tips at the moment when it is increasing most rapidly?(Putnam Exam Question) Answer: Making assumption that both the hands moves at constant angular velocities, the answer is ## \sqrt{7} .## But don't you think this assumption is somewhat doubtful and wrong?

Similar threads

  • · Replies 2 ·
Replies
2
Views
2K
  • · Replies 9 ·
Replies
9
Views
1K
  • · Replies 9 ·
Replies
9
Views
1K
  • · Replies 3 ·
Replies
3
Views
2K
  • · Replies 5 ·
Replies
5
Views
2K
Replies
3
Views
2K
  • · Replies 4 ·
Replies
4
Views
1K
Replies
4
Views
3K
Replies
9
Views
2K
  • · Replies 2 ·
Replies
2
Views
1K